Please help me I keep on getting the third box wrong

Please Help Me I Keep On Getting The Third Box Wrong

Answers

Answer 1

Answer:

[tex](2 {a}^{2} + 1)(1 {a}^{2} + 9)[/tex]

Step-by-step explanation:

[tex]2 {a}^{4} + 19 {a}^{2} + 9 \\ 2 {a}^{4} + {a}^{2} + 18{a}^{2} + 9 \\ {a}^{2} (2 {a}^{2} + 1) + 9(2 {a}^{2} + 1) \\ (2 {a}^{2} + 1)( {a}^{2} + 9) \\ [/tex]


Related Questions

Help!!!
Find the domain of the function. Write the answer in interval notation.

Answers

Answer:

A

Step-by-step explanation:

Given

f(x) = [tex]\frac{2}{3x+2}[/tex]

The denominator cannot be zero as this would make f(x) undefined.

Equating the denominator to zero and solving gives the value that x cannot be.

3x + 2 = 0 ⇒ 3x = - 2 ⇒ x = - [tex]\frac{2}{3}[/tex] ← excluded value

Then

domain is ( - ∞ , - [tex]\frac{2}{3}[/tex] ) U ( - [tex]\frac{2}{3}[/tex], ∞ ) → A

If anyone can help with this ill mark Brainly

Answers

Answer:

In this case, we can do substitution.

Step-by-step explanation:

For the first one, (s - t)(x) = ((x - 5) - 4x^2)(x) = x^2 - 21x

For the second one, (s*t)(x) = ((x - 5) *4x^2)(x) = 4x^4 - 20x^3

And for the last one,  (s+t)(-2) = ((x - 5) + 4x^2)(-2) = -8x^2 - 2x + 10

Hope your happy with the answer :)

Can someone help with problems fivethroughseven

Answers

Answer:

5) 15120

6) 11880

7) 336

Step-by-step explanation:

The formula for permutation where mPn is m!/(m-n)!

Applying this to question 5, we get 9!/4!, which is 15120.

For question 6, we get 12!/8!, which is 11880.

For question 7, we get 8!/5!, which is 336.

Drag


the yellow point until an accurate "height" of the triangle is drawn. Afterwards, fill out the empty boxes below to determine the area of the triangle.

Answers

Answer:

I don't see a yellow point

the red line below is perpendicular to which of the following

Answers

The red line is perpendicular to the y-axis.

These points are linear.
Find the slope.
x-3 -2 -1 0 1/2
y-3 -2 -1 0 1 2
slope = [?]

Answers

Answer:

Slope = 4

Step-by-step explanation:

The slope of a linear equation can be calculated as;

m = y2-y1/x2-x1

What we have to do here is to select any two points ;

(x1,y1) = (3,8)

(x2,y2) = (6,20)

m = (20-8)/(6-3) = 12/3 = 4

Which of the following is most likely the next step in the series?
А.
B.
C.
D.

Answers

Answer:

D

Step-by-step explanation:

The answer is D because the pattern adds one angle every one shape. Ex. Triangle, Rectangle, Pentagon, Hexagon

Match the base to the corresponding height.

Answers

Answer:

I can't see the picture

Step-by-step explanation:

SORRY :(

Please helpppp
Find h

Answers

Answer:

the square root of 85 (so 85 is the answer)

Step-by-step explanation:

You can use the pythagorean theorem to find the height. So the sides of the triangle would contain the radius (4/2 = 2) and 9.

so, 2 squared plus 9 squared = 85

I hope this helps!

Solve for the questions (both of them) and label you answers for which question

Answers

Jjbbh I don’t under stand

If Wade has 2 times as many dimes as quarters and they have a combined value of 270 cents, how many of each coin does he have?

Answers

Answer:

Step-by-step explanation:

If he has twice the number of dimes as quarters, then obviously he has more dimes than quarters. The expression that represents that is

d = 2q

That relates the NUMBER of coins; now we need one that relates the VALUE which is a dollars and cents thing. We know that the combined value of the coins is $2.70. The expression that represents this is

.1d + .25q = 2.70 because dimes are worth .10 and quarters are worth .25

Subbing the first equation into the second gives us

.1(2q) + .25q = 2.70 and

.2q + .25q = 2.70 and

.45q = 2.70 so

q = 6

This means he has 6 quarters. If the umber of dimes is twice as much, then d = 2(6) and d = 12.

He has 6 quarters and 12 dimes

PLEASE HELP ASAP WHAT IS THE ANSWER TO THIS QUADRATIC EQUATIONNNN

Answers

Answer:

pls i don't know

The answer is A. 2

This is because the stretch of the parabola is elongated by 2!

Which number produces an irrational number when multiplied by 3/4

Answers

any irrational number would do, like pi or root(2)

Find the value of x from the following given figures.​

Answers

solution :-

here,

We know that interior opposite angles are equal.

So,

110° = 50° + x (being interior opposite angles)

110° - 50° = x

60° = x

the value of x =60°

hope it is helpful to you ☺️

what do you understand by socialization​

Answers

Socialization is the lifelong process through which people learn the values and norms of a given society.

Find the equation of a line that is perpendicular to y = -3x – 1 and passes through the point
(3,2).
Give your answer in the form y = mx + b.

Answers

the answer is:
y = 1/3x + 1
I think the answer is y= ⅓ + 1

Y=2/7 -7 y=-x+2 What is the solution for this system of equations?​

Answers

Answer:

(61/7, -47/7)

Step-by-step explanation:

Given: y=2/7 -7

           y=-x+2

Rewrite the top equation ( y=2/7 -7

y = -47/7

y = -x + 2

Because both equations are equal to y, we can rewrite it again.

-47/7 = -x + 2

Add x to both sides

x + (-47/7) = 2

Add 47/7 to both sides.

x = 47/7 + 2/1

x = 61/7

As stated in the beginning, y is equal to -47/7

Hope you understand!

Shirley buys fiction books for $20 each, and then marks up by 25% to
resell. What is the markup in dollars?

Answers

Answer:

$5

Step-by-step explanation:

Find the markup by finding 25% of 20:

20(0.25)

= 5

So, the markup is $5

If f(x) = -3 3 and g(x)= 4x2 + 2x - 4, find (f +g)(x).

Answers

Answer:

A. 4x² + 9x/4 - 7

General Formulas and Concepts:

Pre-Algebra

Order of Operations: BPEMDAS

Brackets Parenthesis Exponents Multiplication Division Addition Subtraction Left to Right

Algebra I

Terms/CoefficientsFunctionsFunction Notation

Step-by-step explanation:

Step 1: Define

Identify

f(x) = x/4 - 3

g(x) = 4x² + 2x - 4

(f + g)(x) is f(x) + g(x)

Step 2: Find

Substitute in functions:                                                                                     (f + g)(x) = x/4 - 3 + 4x² + 2x - 4Combine like terms:                                                                                         (f + g)(x) = 4x² + 9x/4 - 7

PLS HELP ASAP, I need it in 10 mins. I GIVE 15 PTS !!!! if v1 = (3,-4) and v2 = (2,6) then v1*v2 is equal to which of the following?
A. 30
B. (-12, -24)
C. (6,-24)
D. -18

Answers

It is C. Good luck on ur test!!!!

v₁·v₂ is -18 which is correct option(D)

What are Arithmetic operations?

Arithmetic operations can also be specified by the subtract, divide, and multiply built-in functions.

The operator that perform arithmetic operation are called arithmetic operators .

Operators which let do basic mathematical calculations

+ Addition operation : Adds values on either side of the operator.

For example 4 + 2 = 6

- Subtraction operation : Subtracts right hand operand from left hand operand.

for example 4 -2 = 2

* Multiplication operation : Multiplies values on either side of the operator

For example 4*2 = 8

/ Division operation : Divides left hand operand by right hand operand

For example 4/2 = 2

Given that,

v1 = (3,-4) and v2 = (2,6)

To determine v₁·v₂

v₁·v₂ = 3·(2) - 4·(6)

v₁·v₂ = 6 - 24

v₁·v₂ = -18

Hence, the v₁·v₂ is -18.

Learn more about Arithmetic operations here:

brainly.com/question/25834626

#SPJ2

Graph the solution of the inequality 3/7(35x-14)<_ 21x/2+3

Answers

Answer:

You'll have a closed circle at x = 2, and shading to the left

See the diagram below

=========================================================

Explanation:

The fractions here are 3/7 and 21/2. The denominators of which are 7 and 2 respectively. The LCD is 7*2 = 14.

If we multiply both sides by 14, then this will clear out the denominators and make the fractions go away.

14*(3/7) = (14*3)/7 = 42/7 = 814*(21/2) = (14*21)/2 = 294/2 = 147

So if we multiplied both sides by 14, then we have these steps

[tex]\frac{3}{7}(35x-14) \le \frac{21x}{2}+3\\\\14*\frac{3}{7}(35x-14) \le 14*\left(\frac{21x}{2}+3\right)\\\\14*\frac{3}{7}(35x-14) \le 14*\left(\frac{21x}{2}\right)+14*\left(3\right)\\\\6(35x-14) \le 147x+42\\\\[/tex]

--------------------------

Let's isolate x

[tex]6(35x-14) \le 147x+42\\\\6(35x)+6(-14) \le 147x+42\\\\210x-84 \le 147x+42\\\\210x-147x \le 42+84\\\\63x \le 126\\\\x \le 126/63\\\\x \le 2\\\\[/tex]

The graph of this will consist of a closed or filled in circle at x = 2. We shade to the left to represent numbers smaller than 2.

So either x = 2 or x < 2.

If we used an open hole at 2, then we wouldn't be including 2 (but we want to include this endpoint).

See the diagram below.

Find mCFB

Help me please

Answers

Answer:

A

Step-by-step explanation:

<CPA = 50o                     Given

<FPB = 50o                      Vertically opposite a 50 degree angle

<CPF = 90o                      Given

<CPB = <FPB + <CPF       Given or found

<CPB = 50 + 90                Substitute

<CPB = 140o

URGENT PLS ANSWER QUICKLY​

Answers

Answer:

9th is 44 mark me as brainlist

Answer:

Question 9 is perimeter

Which number completes the system of linear
inequalities represented by the graph?
y> 2x – 2 and x + 4y > _____

Answers

The system of linear inequalties that is represented by the graph is formed by the expressions y > 2 · x - 2 and x + 4 · y > - 12.

What system of inequalities generate the graph seen in the image?

Herein we have two inequalities generated by two linear functions whose form is described below:

f(x, y) > a (blue region)      (1)

g(x, y) > b (red region)      (2)

By direct comparison, we find that the limit of the red region is represented by the inequality y > 2 · x - 2. From the equation of the line we can derive an expression for the limit of the blue region:

Slope

m = [- 4 - (- 3)]/(4 - 0)

m = - 1/4

Intercept

b = y - m · x

b = - 3 - (- 1/4) · 0

b = - 3

Then, the inequation is represented by the linear equation:

y > - (1/4) · x - 3

4 · y > - x - 12

x + 4 · y > - 12

The system of linear inequalties that is represented by the graph is formed by the expressions y > 2 · x - 2 and x + 4 · y > - 12.

To learn more on inequalities: https://brainly.com/question/20383699

#SPJ1

Marc puts 28 liters of water into a tub and 6 liters of water into a bucket. Then, he pours both containers into a tank.

Answers

What is the question asking for?

Answer:

34

Step-by-step explanation:

A line with a slope of 3 passes through the point (-1, 2).

Write an equation for this line in point-slope form.

Answers

Answer:

Step-by-step explanation:

Slope = m = 3

(x₁ , y₁) = (-1 , 2)

Point slope form: y - y₁ = m(x - x₁)

y - 2 = 3(x - [-1] )

y - 2 = 3(x + 1)

y - 2 = 3*x + 3*1

y - 2 = 3x + 3

    y = 3x + 3 + 2

    y = 3x + 5

Ava’s gross pay is $2,500 per month. Her deductions include the following:

Federal income tax $290

State income tax $85

Social Security $112

Medicare $35

What is Ava’s net pay each month?

Answers

$1,978

Explanation:

Gross pay is what’s made before taxes and necessary deductions, net pay is after taxes and necessary deductions. So we minus the deductions to find the net pay. The equation is 2,500 - 290 - 85 - 112 - 35 = ? . The answer is $1,978.

I hope this helps. Please mark me the Brainliest, it’s not necessary but I put time and effort into every answer and I would appreciate it greatly. Have a great day, stay safe and stay healthy ! :)

CAN SOMEBODY PLEASE HELP MEEEE

Answers

Answer:

105.

Step-by-step explanation:

.

​When CP = Rs 1250 and SP = Rs 1500, find profit or loss percent ?

Answers

Answer:

profit=20%

Step-by-step explanation:

profit%=sp-cp/cp×100%

=1500-1250/1250×100

=250/1250×100

=20%

Solve for x and y:
x – 3y = -8
3x + 2y = 31
Select one:
O a. (-11,-1)
O b. (11,1)
o c. (5,8)
O d. (7,5)

Answers

Answer:

O d. (7,5)

Step-by-step explanation:

x – 3y = -8 (1)

3x + 2y = 31 (2)

From (1)

x = -8 + 3y

Substitute x = -8 + 3y into (2)

3x + 2y = 31 (2)

3(-8 + 3y) + 2y = 31

-24 + 9y + 2y = 31

-24 + 11y = 31

11y = 31 + 24

11y = 55

y = 55/11

y = 5

Substitute y = 5 into (1)

x – 3y = -8 (1)

x - 3(5) = -8

x - 15 = -8

x = -8 + 15

x = 7

(x, y) = (7, 5)

Other Questions
Which of the following are exterior angles? What would you expect to observe when Br2 reacts with 2-butyne? 1.what is a computer?2.what is input device?3.what is output device?4.what is system unit? The leading causes of impaired, or polluted, surface waters in the United States originate from both point and nonpoint sources. Arrange the causes of pollution from most common to least common according to data from 2012.a. pH changesb. sedimentation c. nitrogen and phosphorus d. hypoxia e. pesticides f. pathogensp g. mercury h. dissolved metals (excluding mercury) i. PCBs (industrial pollutant)1. Most common causes 2. Least common causes three condensers are connected in series across a 150 volt supply, the voltages across them are 40,50 and 60 volts respectively, and the charge on each condenser is 610^-8 c.calculate (a) the capacitance of each condenser (b)the effective capacitance of the combination A government bond issued in France has a coupon rate of 5% (paid annually) and a face value of 100 euros, and it matures in 5 years. Calculate the price of the bond (in euros) if the yield to maturity is 3.5%. Help me plz I cant figure it out Helpppppp and explain toooo thankyouuuu Which of the following pieces of legislation was also known as the Wager Act? Thinking Critically and Solving ProblemsWhich conclusion is supported by the graph?Food Spending and Percent of IncomeSpent on Food16,00014,000Spending (eftaris)Percent (right acis)12.000-10,000People in the highest quintile spend aboutOA)$4,000 on food or about 30% of their income.People in the lowest quintile spend half asB)much on food as people in the highest quintile.As their incomes rise, people spend a greaterCportion of their total income on food.As their incomes rise, people spend more onD) food, but it represents a smaller portion oftheir income.Average annual food spending explain the chemical reaction which person is not an american citizen a. Ofelia who was born in Argentina but is working for a year in new york b. Rae who has one american parent and one thai parent c. Afua who was born in Nigeria but has been through a naturalization ceremony d. Desmond whose Irish parents were living in Boston when he was born A company reported the following asset and liability balances at the end of 2015 and 2016: 2015 2016 Assets $150,000 $180,000 Liabilities $70,000 $80,000 If the company paid dividends totaling $5,000, what is the amount of net income for 2016?A. $20,000.B. $105,000.C. $80,000.D. $25,000. I never thought of that these statements alone are notsidered for a gradeAnswer the questions) belowA former patient comes into the dental office and states that she has recently been diagnosed with skin cancer. She demands her xrays and treatment record and claims that she was exposed to unnecessary radiatduring her visits How should the dental assistant respond? Can the dental assistant be held responsible? Determine the dimensions of the rectangle of largest area that can be inscribed in a semicircle of radius 4 Need help on this problem RecursionexerciseCOP 3502; Summer2021We have gone through many examples in the class and those examples are available in the slides and uploaded codes. Try to test those codes and modify as you wish for getting more clarification.In additiontry the following:---------------------------------------------------------------------------------------------------------------------------------------1.What would be the output of the following recursive function if we call rec2(5) A committee that consists of five members are to be chosen from 6 boys and 5 girls. Find the number of different committees that can be formed if only two boys are selected Tema libreContesta a la siguiente pregunta escribiendo dos oracionesutilizando el subjuntivo: Qu haras si pudieras detener eltiempo? Besides the cost of an education what additional expenses must students pay check all that apply